Use quadratic formula to solve a function

Click For Summary
The discussion centers on using the quadratic formula to solve for Lambda1 and Lambda2 in the equation L-1(1-(Lambda1+Lambda2)L+Lambda1Lambda2L2)Ft. A participant attempted to isolate L but concluded that L1 equals Lambda1 and L2 equals Lambda2, leading to a different equation upon substitution. However, it was pointed out that the original expression lacks an equality sign, meaning it is not a valid equation to solve. The participant is seeking clarification on their approach and whether they are missing a crucial step. The conversation emphasizes the necessity of having a proper equation to apply the quadratic formula effectively.
Kinetica
Messages
83
Reaction score
0

Homework Statement



L-1(1-(Lambda1+Lambda2)L+Lambda1Lambda2L2)Ft

Use quadratic formula to solve for Lambda1 and Lambda2


The Attempt at a Solution



I took the equation inside the brackets and solved for L. It turned out that
L1=Lambda1 and L2=Lambda2

But when I plugged those findings back, I got a slightly different equation. Weird...

Anyway, is that a right approach or am I missing something? You can see my full solution in the attachment.

Thanks A LOT!
 

Attachments

Physics news on Phys.org
Kinetica said:

Homework Statement



L-1(1-(Lambda1+Lambda2)L+Lambda1Lambda2L2)Ft

Use quadratic formula to solve for Lambda1 and Lambda2
What you have is not an equation, so you can't solve for anything.
Kinetica said:

The Attempt at a Solution



I took the equation inside the brackets and solved for L.
An equation has an = in it. That's how you know it's an equation. You don't have an equation.
Kinetica said:
It turned out that
L1=Lambda1 and L2=Lambda2

But when I plugged those findings back, I got a slightly different equation. Weird...

Anyway, is that a right approach or am I missing something? You can see my full solution in the attachment.

Thanks A LOT!
 
Question: A clock's minute hand has length 4 and its hour hand has length 3. What is the distance between the tips at the moment when it is increasing most rapidly?(Putnam Exam Question) Answer: Making assumption that both the hands moves at constant angular velocities, the answer is ## \sqrt{7} .## But don't you think this assumption is somewhat doubtful and wrong?

Similar threads

Replies
4
Views
1K
  • · Replies 1 ·
Replies
1
Views
1K
  • · Replies 13 ·
Replies
13
Views
3K
Replies
3
Views
1K
  • · Replies 11 ·
Replies
11
Views
2K
  • · Replies 9 ·
Replies
9
Views
2K
  • · Replies 22 ·
Replies
22
Views
3K
Replies
1
Views
2K
  • · Replies 1 ·
Replies
1
Views
4K
Replies
2
Views
2K